0% found this document useful (0 votes)
18 views

Class1 - Statics

The document discusses a physics class called PhysicsWOOT that covers topics like momentum, energy, collisions, and vectors. It provides an overview of the class including details like dates, times, sections, and instructors. It also includes a transcript of the first class where the instructor reviews concepts like static equilibrium and introduces stress and strain.

Uploaded by

AARUSH DESHPANDE
Copyright
© © All Rights Reserved
We take content rights seriously. If you suspect this is your content, claim it here.
Available Formats
Download as PDF, TXT or read online on Scribd
0% found this document useful (0 votes)
18 views

Class1 - Statics

The document discusses a physics class called PhysicsWOOT that covers topics like momentum, energy, collisions, and vectors. It provides an overview of the class including details like dates, times, sections, and instructors. It also includes a transcript of the first class where the instructor reviews concepts like static equilibrium and introduces stress and strain.

Uploaded by

AARUSH DESHPANDE
Copyright
© © All Rights Reserved
We take content rights seriously. If you suspect this is your content, claim it here.
Available Formats
Download as PDF, TXT or read online on Scribd
You are on page 1/ 54

PhysicsWOOT (2751)

AoPS Staff

Tuesday
Aug 24, 2021 - Apr 12, 2022
7:30 - 9:30 PM ET (4:30 - 6:30 PM PT)

Overview
Lesson 1 (Aug 31) Class Transcript - Static Equilibria, Stress, and Strain
< Go back to the class overview page
Copyright © AoPS Incorporated. This page is copyrighted material. You can view and print this page for your own use, but you
cannot share the contents of this file with others.
Display all student messages • Show few student messages • Hide student messages
meichenlaub 2021-08-31 19:30:45
PhysicsWOOT: Momentum, Energy, Collisions, and Vectors

meichenlaub 2021-08-31 19:30:50


PhysicsWOOT lectures will start tonight! They'll be roughly every other Tuesday. I hope you read the appropriate sections of your
book already, because we're going to jump right into some serious physics.

meichenlaub 2021-08-31 19:31:00


(Handout, sorry!)

meichenlaub 2021-08-31 19:31:07


Because we have so many students enrolled in PhysicsWOOT, we have divided the class into sections. All sections will be taught at
the same time, with the same material. You don't have to worry about being in the right section, because you will be placed in the
correct classroom section automatically. We will keep the class split this way for most of the course.

meichenlaub 2021-08-31 19:31:19


Because we have so many students enrolled in physicsWOOT, we have divided the class into sections. All sections will be taught at
the same time, with the same material. You don't have to worry about being in the right section, because you will be placed in the
correct classroom section automatically. We will keep the class split this way for most of the course.

meichenlaub 2021-08-31 19:31:35


Also, because this is a 2-hour-long class, and 2 hours is a long time, we'll take a 5-minute break roughly halfway through.

meichenlaub 2021-08-31 19:31:45


If you miss a lecture, or would like to review the material, you can see the transcripts of the lectures on the physicsWOOT Home
Page.

meichenlaub 2021-08-31 19:31:49


Let me go over one brief reminder of how the classroom works:

meichenlaub 2021-08-31 19:31:54


The classroom is moderated. This means that the messages you type will come to the instructors rather than going directly into
the room. The instructors will choose some of the messages to share with all of the students.

meichenlaub 2021-08-31 19:32:02


Also, sometimes after asking a question, the instructor will wait a couple minutes before proceeding, giving many students in the
class a chance to respond. Take the time to make sure your responses are clear, and include your reasoning in your responses.

meichenlaub 2021-08-31 19:32:11


My name is Mark Eichenlaub and I am the instructor for this course.

meichenlaub 2021-08-31 19:32:15


Mark joined AoPS in 2018 after completing a PhD in physics (focusing on physics education research) at the University of
Maryland. There, he also prepped the U.S. Physics Team and attended IPhO as a coach. He has been teaching physics and
astronomy classes since undergrad, and is always impressed by the creativity and resilience students show when they have a
Typesetting math: 18%
great problem to work on. If you start explaining a proof to him and he happens to see how it works half way through, he'll spend
the rest of the time flailing his arms around conducting the final steps like Willem Dafoe in Boondock Saints. On weekends you
might find him exploring the miles of trails San Diego has to offer.

meichenlaub 2021-08-31 19:32:26


Your assistants for this course will be Nikola Ivanov Yanakiev (Nikola2024) and Prathyush Prasanth Poduval (pp_poduval).

meichenlaub 2021-08-31 19:32:33


Nikola2024: Nikola joined AoPS in 2019. He has always been very fond of both Mathematics and Science and started participating
in math contests at the very early age of 8. After finishing fourth grade he transferred to one of the best Mathematics high schools
in Bulgaria – Sofia High School of Mathematics. There he found his love for Physics and started to take part in Physics
competitions as well. His team took second place in the 9th International Festival of Young Mathematicians, and he became a
scholar at the third and final round of the National Physics Olympiad. Nikola also won the opportunity to represent Bulgaria at the
S’Cool LAB Summer Camp at CERN in 2018, which further pushed him to pursue physics at Cambridge.

meichenlaub 2021-08-31 19:32:40


pp_poduval: Prathyush Poduval is currently an undergraduate majoring in theoretical physics at the Indian Institute of Science,
Bangalore. Though he's mainly interested in physics, circumstances allowed him to explore mathematics in greater detail.
Prathyush finds problem solving a fun recreational exercise that allows him to relax. He has participated in the Ross math program
both as a student and counsellor which has played a profound role in shaping the way he thinks about new concepts and problems.
He's also an avid amatuer astronomer, and loves taking his telescopes out when it's not cloudy!

pp_poduval 2021-08-31 19:32:50


Hello everyone!

meichenlaub 2021-08-31 19:32:55


Please feel free to ask us questions at any time during class. Instructors and assistants have the ability to send private messages
in the classroom. Sometimes we will use these to respond to your comments. The instructor or teaching assistant may initiate a
private conversation with you that will open in a separate window.

meichenlaub 2021-08-31 19:33:09


I want to remind you about the difficulty of the material in WOOT, which we talked about last week.

meichenlaub 2021-08-31 19:33:12


Please keep in mind during the course that some of you are taking WOOT for the first time, and others may have taken WOOT
before. Because we have a wide range of students in the course, there is often a wide range of difficulty in the material, in both the
classes and the homework.

meichenlaub 2021-08-31 19:33:24


In particular, we do not expect you to solve every problem, especially if you are a first-time WOOT student! AoPS courses are
specifically designed to challenge even the strongest students, and WOOT is certainly no exception. We want you to try to solve as
much as you can, but "getting 100%" is certainly not the goal.

meichenlaub 2021-08-31 19:33:36


Conversely, if you've taken WOOT before: even though the topics in this year's version of WOOT may be new, you may find the
difficulty of some the classes not as challenging as it was when you first took WOOT. That's because (ideally) you've improved your
problem-solving skills! But keep in mind that many of your fellow students are first-timers here. Also, we still expect that the
harder material in both the classes and the HW will challenge you!

meichenlaub 2021-08-31 19:33:48


This week, we'll start by reviewing statics: situations where systems are in equilibrium. We'll practice a couple of different
perspectives on these problems, then move on to a topic that will be new to many of you: stress and strain.

meichenlaub 2021-08-31 19:33:59


These concepts were introduced briefly with some examples on the handout. It's important to read the handout before class so
you're prepared when we start solving problems together.

meichenlaub 2021-08-31 19:34:07


Statics and Forces

meichenlaub 2021-08-31 19:34:16


Suppose an object isn't moving. Then of course there's no net force on it. Let's do a couple of quick problems of this nature.
Typesetting math: 18%
meichenlaub 2021-08-31 19:34:25
Suppose a block of mass m sits on an plane inclined at an angle θ. The coefficient of friction between the block and the plane is μ.
A force F is applied to the block, pushing it sideways, in the direction parallel to the plane, but perpendicular to the direction
"uphill".

meichenlaub 2021-08-31 19:34:36

meichenlaub 2021-08-31 19:35:10


(That white patch on the wedge is the block; I'm not sure why it isn't rendering completely correctly.)

meichenlaub 2021-08-31 19:35:27


What is the maximum F such that the block will not move?

meichenlaub 2021-08-31 19:35:40


By the way, many Olympiad problems don't bother distinguishing between static and kinetic friction coefficients, although some
do. If the problem doesn't say, you should assume the static and kinetic friction coefficients are the same.

meichenlaub 2021-08-31 19:35:53


You should be familiar with a very similar problem to this, which allows us to jump ahead some in our analysis.

meichenlaub 2021-08-31 19:35:59


For example, what's the normal force on the block, assuming it's not moving?

PhysKid11 2021-08-31 19:36:36


mgcos(theta)

sienna42 2021-08-31 19:36:36


mgcos theta

A-Aban 2021-08-31 19:36:36


mgcos(theta)

mathtiger6 2021-08-31 19:36:36


mgcosθ

olive0827 2021-08-31 19:36:36


mg*cos(theta)

HappiHippo 2021-08-31 19:36:36


mgcosθ

Z_Lu 2021-08-31 19:36:36


cos(thetha)mg

mathisawesome01 2021-08-31 19:36:36


mgcosθ

jspiepuppy 2021-08-31 19:36:36


mgcos(theta)

Siddhartha.basu 2021-08-31 19:36:36


mgcostheta
Typesetting math: 18%
aj1745 2021-08-31 19:36:36
mgcos(theta)

DGA_Student_1 2021-08-31 19:36:36


mgcos(θ)

theallpro1 2021-08-31 19:36:36


mgcosθ

olive0827 2021-08-31 19:36:36


mgcos(θ)

Yufanwang 2021-08-31 19:36:36


mgcosθ

Nivek 2021-08-31 19:36:36


mg cos theta

rohitk14 2021-08-31 19:36:36


mgcos(theta)

meichenlaub 2021-08-31 19:36:47


The normal force is

meichenlaub 2021-08-31 19:36:54

N = mgcosθ.

meichenlaub 2021-08-31 19:37:04


(This was derived in the handout. It's important to go over the handouts before class so you are caught up with the material there
as we solve problems together.)

meichenlaub 2021-08-31 19:37:17


And what's the friction force if F = 0 (i.e. we're not pushing it sideways at all) and the block is just sitting on the plane?

rohitk14 2021-08-31 19:38:18


mgsin(theta)

mathtiger6 2021-08-31 19:38:18


mgsinθ

Shadow_Heist 2021-08-31 19:38:18


mgsinθ

mathisawesome01 2021-08-31 19:38:18


mgsinθ

Pleaseletmewin 2021-08-31 19:38:18


mgsinθ

DGA_Student_1 2021-08-31 19:38:18


mgsinθ up the incline

theallpro1 2021-08-31 19:38:18


mgsinθ

jszumow 2021-08-31 19:38:18


mg*sin(theta)

Hershey806 2021-08-31 19:38:18


mg sin theta

A-Aban 2021-08-31 19:38:18


mgsin(theta)

PhysKid11
Typesetting 2021-08-31
math: 18% 19:38:18
mg sin(theta)
EL2024 2021-08-31 19:38:18
mgsin(theta)?

meichenlaub 2021-08-31 19:38:25


It's not μN. The formula for static friction is actually F friction ≤ μN. So we can't use this law to figure out the friction force.

meichenlaub 2021-08-31 19:38:35


Instead, we recognize that the friction force in this case is canceling the component of gravitational force that points down the
plane. From previous problems, we recognize that as

meichenlaub 2021-08-31 19:38:41

F friction , no push = mgsinθ.

meichenlaub 2021-08-31 19:38:56


That's the friction force when we don't push on the block. Suppose we do push on the block. Then the friction force changes.

meichenlaub 2021-08-31 19:39:05


The problem says we push sideways with the force F. So describe how the total friction force changes when we do that (assuming
the block still doesn't move)?

jspiepuppy 2021-08-31 19:40:14


the total friction force increases

Shadow_Heist 2021-08-31 19:40:14


It increases

mathtiger6 2021-08-31 19:40:14


it increases and changes angle

HappiHippo 2021-08-31 19:40:14


the friction force increases to counteract F

EthanTLee23 2021-08-31 19:40:14


The friction force increases

ethanchen323 2021-08-31 19:40:14


theres also a friction force opposing the direction of F

DGA_Student_1 2021-08-31 19:40:14


tilts sideways and increases in magnitude

A-Aban 2021-08-31 19:40:14


another component of friction opposite of F?

Z_Lu 2021-08-31 19:40:14


The friction for increases until it reached the maximum, at which then it will move

theallpro1 2021-08-31 19:40:14


the friction we found above will be the same but there will also be a component of the friction force that is pointing opposite to F

Siddhartha.basu 2021-08-31 19:40:14


The friction force now has to oppose the block sideways, which is how it changes.

taniaban 2021-08-31 19:40:14


it increases (vector sum of F and mg sin theta)

meichenlaub 2021-08-31 19:40:19


The friction force gains a component that cancels the force F. So friction force increases in magnitude and has a component
pointing across the inclined plane, in the opposite direction as F.

PhysKid11 2021-08-31 19:41:34

√ F 2f + F 2 math: 18%
Typesetting
DGA_Student_1 2021-08-31 19:41:34
2
√ Ff +F 2

vkrishnanaz 2021-08-31 19:41:34


sqrt(Ff^2 + F^2)

mathtiger6 2021-08-31 19:41:34

√F 2
+ m g sin 2θ
2 2

sienna42 2021-08-31 19:41:34

√F 2
+ m g cos 2θ
2 2

bfan0805 2021-08-31 19:41:34


2
√ Ff +F 2

HappiHippo 2021-08-31 19:41:34


2
√ Ff +F 2

Hershey806 2021-08-31 19:41:34

F 2f
√ + F2

Shadow_Heist 2021-08-31 19:41:34

√ F 2f +F 2

mathisawesome01 2021-08-31 19:41:34

√ F 2f + F2

aj1745 2021-08-31 19:41:34


sqrt(F^2+Ff^2)

Zian2006 2021-08-31 19:41:34

F 2f
√ +F 2

EthanTLee23 2021-08-31 19:41:34


sqrt(F^2+(Ff)^2)

sienna42 2021-08-31 19:41:34


\sqrt{F^2 + F_f^2}

meichenlaub 2021-08-31 19:41:42


The two components of the frictional force are perpendicular to each other, so we can find their magnitude with the Pythagorean
theorem. It comes to

meichenlaub 2021-08-31 19:41:45

2
F friction total =
√F f
+ F 2.

meichenlaub 2021-08-31 19:41:52


Substituting in our earlier conclusion about F f = mgsinθ, we have

F friction total = √(mgsinθ) 2 + F 2.

meichenlaub 2021-08-31 19:42:00


And what's
Typesetting the18%
math: maximum F in terms of the variables given in the problem?
meichenlaub 2021-08-31 19:42:01
(You'll have to do a few steps for this, so work it out on paper. Make sure you have paper and a calculator ready during each class!)

Pleaseletmewin 2021-08-31 19:44:20

F ≤ mg √ μ 2cos 2θ − sin 2θ

mathtiger6 2021-08-31 19:44:20


F ≤ mg μ cos 2θ − sin 2θ
2

sienna42 2021-08-31 19:44:20


F = mg μ cos 2θ − sin 2θ
2

mathisawesome01 2021-08-31 19:44:20


F max = mg (μcosθ) − sin 2θ)
2

mathisawesome01 2021-08-31 19:44:20


F max = mg (μcosθ) − sin 2θ
2

Zian2006 2021-08-31 19:44:20


mg μ cos θ − sin 2θ
2 2

Shadow_Heist 2021-08-31 19:44:20


mg μ 2cos 2θ − sin 2θ

bfan0805 2021-08-31 19:44:20


F = mg μ 2cos 2θ − sin 2θ

EthanTLee23 2021-08-31 19:44:20


F=mg*sqrt((μcosθ)^2-(sinθ)^2)

meichenlaub 2021-08-31 19:44:30


The friction force must obey F friction total ≤ μN.

meichenlaub 2021-08-31 19:44:34


Making that substitution and using N = mgcosθ, we have

√(mgsinθ) 2 + F 2 ≤ μmgcosθ.

meichenlaub 2021-08-31 19:44:45


Solving for F, we get


F ≤ mg (μcosθ) 2 − sin 2θ.

meichenlaub 2021-08-31 19:44:55


So the maximum force is


F maximum = mg (μcosθ) 2 − sin 2θ.

meichenlaub 2021-08-31 19:45:02


After we solve a problem, we usually want to analyze our solution to see if it makes sense.

meichenlaub 2021-08-31 19:45:05


In this case, let's test our solution by looking at the special case F maximum = 0.

meichenlaub 2021-08-31 19:45:09


Suppose we
Typesetting have
math: 18% F
maximum = 0. Then what does our solution let us conclude about the relationship between μ and θ ?
mathtiger6 2021-08-31 19:46:24
μ = tanθ

sienna42 2021-08-31 19:46:24


mu cos theta = sin theta

Pleaseletmewin 2021-08-31 19:46:24


μ = tanθ

Zian2006 2021-08-31 19:46:24


μ = tanθ

Hershey806 2021-08-31 19:46:24


μ = tanθ

AOPS81619 2021-08-31 19:46:24


μcosθ = sinθ

EL2024 2021-08-31 19:46:24


u=tan(theta)

sienna42 2021-08-31 19:46:24


mu = tan theta

EthanTLee23 2021-08-31 19:46:24


μ=tanθ

PhysKid11 2021-08-31 19:46:24


mu=tan(theta)

Siddhartha.basu 2021-08-31 19:46:24


μ=tan(theta)

Shadow_Heist 2021-08-31 19:46:24


μ = tanθ

DGA_Student_1 2021-08-31 19:46:24


actually μ = tanθ

taniaban 2021-08-31 19:46:24


μ = tanθ

theallpro1 2021-08-31 19:46:24


tan theta = u

smhgeo422 2021-08-31 19:46:24


μ = tan(θ)

Z_Lu 2021-08-31 19:46:24


$\mu=\tan\theta

meichenlaub 2021-08-31 19:46:30


In this case,


0 = mg (μcosθ) 2 − sin 2θ.

meichenlaub 2021-08-31 19:46:36


Dividing by mg and squaring we get

(μcosθ) 2 − sin 2θ = 0.

meichenlaub 2021-08-31 19:46:42


And this means
Typesetting math: 18%
μcosθ = sinθ,

or

μ = tanθ.

meichenlaub 2021-08-31 19:46:47


Where have you seen that before?

EL2024 2021-08-31 19:47:13


maximum coefficient for incline plane

theallpro1 2021-08-31 19:47:13


simple block+plane+friction slipping angle

Hershey806 2021-08-31 19:47:13


stationary block on inclined plane with friction

bfan0805 2021-08-31 19:47:13


critical angle

sienna42 2021-08-31 19:47:13


minimum mu so that the block doesn't slide down

taniaban 2021-08-31 19:47:13


max value of theta so the block doesn't slip

Latnor 2021-08-31 19:47:13


handout

mathtiger6 2021-08-31 19:47:13


minimum μ for the block to remain at rest by itself

EL2024 2021-08-31 19:47:13


maximum angle

meichenlaub 2021-08-31 19:47:16


Right, on the handout for today's class, we solved a problem with a rock climber standing on a sloped surface, and we found that
the steepest slope they can stand on obeys the same equation.

meichenlaub 2021-08-31 19:47:35


If the rock climber was on the steepest possible slope, then we couldn't put any horizontal force on them, or else they'd slip. So if
F maximum = 0, the two problems are the same, and we ought to get the same answer for them. This makes us a bit more confident
that our answer to this more-complicated problem is correct.

meichenlaub 2021-08-31 19:48:00


Let's do another quick problem on forces.

meichenlaub 2021-08-31 19:48:03


A flat horizontal platform rotates at angular frequency ω. A block sits on the platform a distance r from the axis of rotation. What
is the minimum coefficient of friction needed to keep the block from slipping relative to the platform?

meichenlaub 2021-08-31 19:48:12


The first thing you might notice is that this isn't a statics problem - the block is accelerating.

meichenlaub 2021-08-31 19:48:15


But what can we do to turn it into a statics problem?

DGA_Student_1 2021-08-31 19:49:27


switch into rotating frame

Hershey806 2021-08-31 19:49:27


look at it in
Typesetting the18%
math: rotating frame
bfan0805 2021-08-31 19:49:27
change reference frames

HappiHippo 2021-08-31 19:49:27


look at the problem in the frame of the platform

sienna42 2021-08-31 19:49:27


transfer into the reference frame of the block

Zian2006 2021-08-31 19:49:27


Centrifugal force

Shadow_Heist 2021-08-31 19:49:27


take a non-inertial RF

mathtiger6 2021-08-31 19:49:27


rotating frame of reference

theallpro1 2021-08-31 19:49:27


view the problem in the frame of the platform

jspiepuppy 2021-08-31 19:49:27


change the frame of reference to that of the platform

ethanchen323 2021-08-31 19:49:27


view from the platform's frame and add fictitious forces

Yufanwang 2021-08-31 19:49:27


view it from the reference frame of the platform

Shadow_Heist 2021-08-31 19:49:27


include the "centrifugal force"

meichenlaub 2021-08-31 19:49:41


We can adopt a rotating reference frame! These means we imagine ourselves rotating along with the platform. Then from our point
of view, the block is stationary.

meichenlaub 2021-08-31 19:50:01


In this reference frame, there's a centrifugal force of magnitude

meichenlaub 2021-08-31 19:50:03

F c = mω 2r

meichenlaub 2021-08-31 19:50:20


pushing the block away from the axis of rotation.

meichenlaub 2021-08-31 19:50:24


Because the centrifugal force is proportional to mass, just like gravitational force is, we can think of this as if gravitational force
has been modified to point at a new angle θ relative to the old gravitational force.

meichenlaub 2021-08-31 19:50:30

Typesetting math: 18%


meichenlaub 2021-08-31 19:50:40
What's an equation for that new angle?

EL2024 2021-08-31 19:51:52


tan(theta)=w^2r/g

mathtiger6 2021-08-31 19:51:52


ω 2r
tanθ = g

mathisawesome01 2021-08-31 19:51:52


ω 2r
tanθ =
g

sienna42 2021-08-31 19:51:52


tan theta = omega^2 r/g

Pleaseletmewin 2021-08-31 19:51:52

θ = arctan
( )ω 2r
g

DGA_Student_1 2021-08-31 19:51:52


rω 2
tanθ = g

PhysKid11 2021-08-31 19:51:52


arctan(w^2r/g)

bfan0805 2021-08-31 19:51:52

θ = tan − 1
( )
mω 2r
mg

Hershey806 2021-08-31 19:51:52


ω 2r
tanθ = g

AOPS81619 2021-08-31 19:51:52


ω 2r
tanθ = g

theallpro1 2021-08-31 19:51:52


2
mω r
tanθ =
mg
Typesetting math: 18%
EthanTLee23 2021-08-31 19:51:52
tanθ = (ω^2*r)/g

pimath 2021-08-31 19:51:52


tan theta = w^2r/g

JohnOfMages 2021-08-31 19:51:52


arctan(mw^2r/mg)

Zian2006 2021-08-31 19:51:52


ω 2r
tan − 1( g )

meichenlaub 2021-08-31 19:51:59


The effective gravitational acceleration has a horizontal component of ω 2r and a vertical component of g. So we have

ω 2r
tanθ = .
g

meichenlaub 2021-08-31 19:52:08


And what's the solution to the problem?

Zian2006 2021-08-31 19:53:18


ω 2r
μ min = g

bfan0805 2021-08-31 19:53:18


ω 2r
μ≥ g

mathisawesome01 2021-08-31 19:53:18


ω 2r
μ min =
g

DGA_Student_1 2021-08-31 19:53:18


rω 2
μ min = g

sienna42 2021-08-31 19:53:18


mu = omega^2 r / g

Hershey806 2021-08-31 19:53:18


ω 2r
μ= g

EthanTLee23 2021-08-31 19:53:18


mu = ω^2*r/g

Pleaseletmewin 2021-08-31 19:53:18


2
μ ≥ tanθ = ω r / g

mathtiger6 2021-08-31 19:53:18


ω 2r
μ ≥ tanθ = g

Shadow_Heist 2021-08-31 19:53:18


ω 2r
μ= g

pimath 2021-08-31 19:53:18


u = tan theta = w^2r/g

meichenlaub 2021-08-31 19:53:23


We already know that things slide when they're on a surface where μ = tanθ and θ is the angle of the surface relative to
gravitational acceleration. That's exactly the situation here, so we have

Typesetting math: 18%


ω 2r
μ minimum = .
g

meichenlaub 2021-08-31 19:53:37


Of course, this isn't the only way to solve this problem and a rotating reference frame isn't necessary, so you might want to revisit
this problem later and solve it from an inertial frame.

meichenlaub 2021-08-31 19:53:47


A suspension bridge consists of a heavy road with mass per unit length λ supported by a massless cable hanging between
supports. Frequent small massless vertical lines connect the cable to the road. Find the shape of the cable.

meichenlaub 2021-08-31 19:53:56


Here's the shape we're interested in:

meichenlaub 2021-08-31 19:53:58

meichenlaub 2021-08-31 19:54:03


(This is the Golden Gate Bridge in San Francisco.)

meichenlaub 2021-08-31 19:54:08


In the handout for this week, we saw an approach to a similar problem where we treat a large piece of the rope as our system.

meichenlaub 2021-08-31 19:54:14


An important idea for this problem is that we can choose whatever subsystem we want and balance the forces on it.

meichenlaub 2021-08-31 19:54:19


In other words, we don't need to look at the entire cable and find the forces on it. We can instead choose any particular piece of
cable. The forces on that piece of cable must sum to zero.

meichenlaub 2021-08-31 19:54:34


In this problem, is there any special point that stands out - perhaps one where one of the components of tension in the cable will
be zero, simplifying the equations for tension?

mathisawesome01 2021-08-31 19:55:45


bottom

vkrishnanaz 2021-08-31 19:55:45


the midpoint

EL2024 2021-08-31 19:55:45


the bottom?

PhysKid11 2021-08-31 19:55:45


the center of the cable

Hershey806 2021-08-31 19:55:45


at the bottom?

mathtiger6 2021-08-31 19:55:45


the bottommost point
Typesetting math: 18%
No_u_ 2021-08-31 19:55:45
the middle

HappiHippo 2021-08-31 19:55:45


at the vertex of the parabolic shape?

EthanTLee23 2021-08-31 19:55:45


bottom?

Z_Lu 2021-08-31 19:55:45


The middle of the bridge would not have any tension in the y direction

Pleaseletmewin 2021-08-31 19:55:45


the bottom of the cable

Hiboy 2021-08-31 19:55:45


Middle point

Yufanwang 2021-08-31 19:55:45


The "horizontal" part of the cable

taniaban 2021-08-31 19:55:45


bottom-most point

JohnOfMages 2021-08-31 19:55:45


the stationary point of the parabola, where mg will be cancelled out by the tensions on both sides.

Latnor 2021-08-31 19:55:45


nvm the middle

Zian2006 2021-08-31 19:55:45


Center point?

jspiepuppy 2021-08-31 19:55:45


the middle vertical line connecting the cable to the road

harinsrikanth 2021-08-31 19:55:45


The lowest point of the cable

theallpro1 2021-08-31 19:55:45


the middle of the cable

meichenlaub 2021-08-31 19:55:52


The center of the cable is a point like that. The cable is horizontal in the center, so the tension is all in the x direction there.

meichenlaub 2021-08-31 19:56:01


So let's take as our system a piece of the cable, stretching from the center of the cable a distance x to the right and y up.

meichenlaub 2021-08-31 19:56:07

meichenlaub 2021-08-31 19:56:11


What forces should we include?

Hershey806 2021-08-31 19:57:31


tension and gravity

PhysKid11
Typesetting 2021-08-31
math: 18% 19:57:31
tension and gravity
Nivek 2021-08-31 19:57:31
the two tensions and gravity

mathisawesome01 2021-08-31 19:57:31


Tension forces at the left and at the right

bfan0805 2021-08-31 19:57:31


tension and gravity

DGA_Student_1 2021-08-31 19:57:31


tension on either end and gravity

A-Aban 2021-08-31 19:57:31


gravity

JohnOfMages 2021-08-31 19:57:31


weight and tension.

taniaban 2021-08-31 19:57:31


tension and gravity

theallpro1 2021-08-31 19:57:31


gravity and tension

AOPS81619 2021-08-31 19:57:31


gravity and tension

Hiboy 2021-08-31 19:57:31


graivty acting on center, 2 tension forces from either ends

mathtiger6 2021-08-31 19:57:31


tension and gravity

YTT 2021-08-31 19:57:31


gravity (definitely)

ethanchen323 2021-08-31 19:57:31


gravity and tensions (right and left along the cable)

EL2024 2021-08-31 19:57:31


tension on ends?

Yufanwang 2021-08-31 19:57:31


Gravitational force, tension

harinsrikanth 2021-08-31 19:57:31


Gravity, tension

EthanTLee23 2021-08-31 19:57:31


weight of the section, tension at the leftmost point, tension at the rightmost point

HappiHippo 2021-08-31 19:57:31


gravity and upward component of tension

jspiepuppy 2021-08-31 19:57:31


tension in the x direction, gravitational force

lasagna 2021-08-31 19:57:31


gravity, tension

Z_Lu 2021-08-31 19:57:31


The force of gravity acting in the middle,the tension force on the right hand side

Zian2006 2021-08-31 19:57:31


Downward
Typesetting force
math: 18%from the bridge, two tensions pulling at the 2 endpoints of the cable
jszumow 2021-08-31 19:57:31
gravity

meichenlaub 2021-08-31 19:57:38


There's a gravitational force downward, a tension force on the right, and a tension force on the left. The tension force on the right
can be broken down into horizontal and vertical components.

meichenlaub 2021-08-31 19:57:41

meichenlaub 2021-08-31 19:58:21


(Those saying "just tension" are also correct. It's not really a gravitational force on the cable; it's a downward tension force from the
small vertical cables, and its magnitude is equal to that of the gravitational force on the road beneath the cable.)

meichenlaub 2021-08-31 19:58:40


Summarize what this diagram tells us about the horizontal component of tension throughout the cable.

mathtiger6 2021-08-31 19:59:53


just tension

Pleaseletmewin 2021-08-31 19:59:53


it is constant

taniaban 2021-08-31 19:59:53


constant

EL2024 2021-08-31 19:59:53


horizontal component says the same

bfan0805 2021-08-31 19:59:53


it must be constant since its not accelerating horizontally

DGA_Student_1 2021-08-31 19:59:53


the horizontal component of the tension is constant

Zian2006 2021-08-31 19:59:53


Equal to T L

mathisawesome01 2021-08-31 19:59:53


The horizontal component of tension is constant throughout the rope.

Hiboy 2021-08-31 19:59:53


Typesetting math: 18%
t_L= T_R;x
EthanTLee23 2021-08-31 19:59:53
The net force must be 0, so TL=TRx

mathtiger6 2021-08-31 19:59:53


horizontal component is equal throughout the cable

Shadow_Heist 2021-08-31 19:59:53


the horizontal component is equal throughout

Latnor 2021-08-31 19:59:53


always the same

PhysKid11 2021-08-31 19:59:53


it is equal all throughout the cable, since horizontal forces must cancel

meichenlaub 2021-08-31 20:00:07


In the horizontal direction, there are two forces: T L to the left and T R ; x to the right. Therefore, T R ; x = T L. This is true at any x, so
the horizontal component of the tension is uniform throughout the cable and equal to T L.

meichenlaub 2021-08-31 20:00:47


("Uniform" means "the same at all points in space" and "constant" means "the same at all times", so it's better to say the horizontal
component of tension is "uniform" than "constant", but people will know what you mean either way.)

meichenlaub 2021-08-31 20:00:57


Again balancing forces, but this time in the vertical direction, we have

T R ; y = mg.

meichenlaub 2021-08-31 20:01:04


What's m and why?

mathtiger6 2021-08-31 20:02:51


m = λx because the bridge is horizontal underneath the cable

Bossman5960 2021-08-31 20:02:51


m = xλ because that's the length times mass per length giving mass

ishaansiyer 2021-08-31 20:02:51


mass of the road since the cable is holding the road up

jspiepuppy 2021-08-31 20:02:51


The mass of the bridge because the piece of cable has to bear the weight of the heavy road

DGA_Student_1 2021-08-31 20:02:51


m = λx

Pleaseletmewin 2021-08-31 20:02:51


m = λx, as this is the mass of the part of the road that the cable is connected to

meichenlaub 2021-08-31 20:03:09


It's λx. A horizontal length x of cable supports the mass of the road beneath it, which is λx.

meichenlaub 2021-08-31 20:03:31


dy
Can you use this to get an equation for in terms of λ, g, x, and T L ?
dx

AOPS81619 2021-08-31 20:05:32


dy λxg
= T
dx L

Zian2006 2021-08-31 20:05:32


λxg
dy / dx = T
L
Typesetting math: 18%
DGA_Student_1 2021-08-31 20:05:32
dy λxg
=
dx TL

harinsrikanth 2021-08-31 20:05:32


λxg / T l

Pleaseletmewin 2021-08-31 20:05:32


dy λxg
=
dx TL

EL2024 2021-08-31 20:05:32


dy/dx=(lambda)xg/(TL)

mathtiger6 2021-08-31 20:05:32


λxg
TL

bfan0805 2021-08-31 20:05:32


λxg
TL

Hershey806 2021-08-31 20:05:32


dy λxg
= T
dx L

EthanTLee23 2021-08-31 20:05:32


dy/dx=λxg/(TL)

mathisawesome01 2021-08-31 20:05:32


dy λxg
We have that = (the angles must be equal, calculated both ways)
dx TL

Bossman5960 2021-08-31 20:05:32


dy xλg
= T
dx L

Hiboy 2021-08-31 20:05:32


Xlambdag/T_L

meichenlaub 2021-08-31 20:05:40


Looking at the right hand side of the cable, the tension must point along the cable, so

meichenlaub 2021-08-31 20:05:44

dy TR ; y λgx
= = .
dx TR ; x TL

meichenlaub 2021-08-31 20:05:50


And what's y(x) ?

Pleaseletmewin 2021-08-31 20:08:24


λgx 2
, which is a quadratic
2T L

mathtiger6 2021-08-31 20:08:24


λgx 2
2T L

Hershey806 2021-08-31 20:08:24


λgx 2
2T L

mathisawesome01 2021-08-31 20:08:24


2
λgx
y(x) = x 0 +
2T
Typesetting math:L18%
Hiboy 2021-08-31 20:08:24
Lambda*g*x^2/(T_L*2)

Zian2006 2021-08-31 20:08:24


λgx 2
+C
2T L

vkrishnanaz 2021-08-31 20:08:24


y(x) = l*gx^2/(2Tl)

EthanTLee23 2021-08-31 20:08:24


y(x)=λ(x^2)g/(2*TL)

DGA_Student_1 2021-08-31 20:08:24


λgx 2
y(x) = 2T
L

HappiHippo 2021-08-31 20:08:24


λg
x2
2T L

Latnor 2021-08-31 20:08:24


λgx 2
2T L

A-Aban 2021-08-31 20:08:24


lamda*g*x^2 / 2TL

taniaban 2021-08-31 20:08:24


λg 2
x
2T L

JohnOfMages 2021-08-31 20:08:24


y(x) = (λgx^2)/2TL + C?

AOPS81619 2021-08-31 20:08:24


λgx λgx 2
y = ∫ T dx = 2T
L L

mathisawesome01 2021-08-31 20:08:24


2
λgx
y(x) = y 0 +
2T L

Bossman5960 2021-08-31 20:08:24


λgx 2
y(x) = 2T ?
L

ethanchen323 2021-08-31 20:08:24


λg 2
y(x) = 2T x
L

No_u_ 2021-08-31 20:08:24


2
λgx / (2T L)

meichenlaub 2021-08-31 20:08:35


We can separate the differential equation, multiplying both sides by dx to get

λgx
dy = dx.
TL

meichenlaub 2021-08-31 20:08:39


Integrating both sides, we have

Typesetting math: 18%



x x λgx
∫ dy = ∫0 dx ′ .
0 TL

meichenlaub 2021-08-31 20:08:45


And performing the integral,

λgx 2
y(x) − y(0) = ,
2T L

or

λgx 2
y(x) = y(0) + .
2T L

meichenlaub 2021-08-31 20:09:01


That shape is a parabola.

meichenlaub 2021-08-31 20:09:09


What could engineers change (while keeping the same model, just adjusting the parameters) to make the parabola flatter?

Zian2006 2021-08-31 20:10:14


Pull more at the ends

DGA_Student_1 2021-08-31 20:10:14


decrease the mass per unit length of the road, increase the horizontal tension

mathtiger6 2021-08-31 20:10:14


make the road less dense

Pleaseletmewin 2021-08-31 20:10:14


decrease λ

jspiepuppy 2021-08-31 20:10:14


the mass per unit length

Hiboy 2021-08-31 20:10:14


The horizontal tenion

mathisawesome01 2021-08-31 20:10:14


λ decreases

EL2024 2021-08-31 20:10:14


lower lambda

A-Aban 2021-08-31 20:10:14


make a lighter road

JohnOfMages 2021-08-31 20:10:14


increase the value of TL so that the coefficient of x^2 is smaller.

Zian2006 2021-08-31 20:10:14


Or making the bridge lighter

mathisawesome01 2021-08-31 20:10:14


T L increases

No_u_ 2021-08-31 20:10:14


lighter bridge

bfan0805 2021-08-31 20:10:14


make λ smaller
Typesetting math: 18%
taniaban 2021-08-31 20:10:14
decreasing λ

Yufanwang 2021-08-31 20:10:14


Make T L greater

meichenlaub 2021-08-31 20:10:22


λg
A flatter parabola means the coefficient would have to be small. So they'd either need to make the bridge lighter or increase
2T L
the tension in the cables.

meichenlaub 2021-08-31 20:10:36


There are two other versions of this problem that you could on on your own:

meichenlaub 2021-08-31 20:10:40


1) What if there's no road, but the cable has mass, so the cable has to support its own weight?

2) Again, the cable supports its own weight, but instead of a fixed-length cable, it's a spring of zero rest length. Then what's the
shape?

meichenlaub 2021-08-31 20:10:48


(These versions are worked out in next year's PhysicsWOOT as well.)

meichenlaub 2021-08-31 20:11:24


A "spring loaded camming device", or "friend" is an aid for rock climbers. Climbers insert one into a crag in the rock and use it to
anchor their ropes.

meichenlaub 2021-08-31 20:11:29

meichenlaub 2021-08-31 20:11:34


The device consists of two cams attached at a common hinge. The cams are free to rotate about the hinge. The cams are designed
so that no matter the width of the crag (within a range), the segment from the point of contact of the cam with the crag to the hinge
is fixed at θ.

meichenlaub
Typesetting math: 18%
2021-08-31 20:12:00
Find the minimum coefficient of friction between the crag and the cam so that the friend will not slip under arbitrarily large loads
W. You can assume the cam is massless.

meichenlaub 2021-08-31 20:12:32


Let's take a single cam as our system. The cams are free to rotate about the hinge, so the cam itself is the system that is rigid. If
we take the entire friend, we won't have a good method to determine how large the horizontal forces, and hence the friction forces,
are.

meichenlaub 2021-08-31 20:12:54


Let's start by looking at the forces. What are the forces on the cam?

EL2024 2021-08-31 20:14:31


normal force with wall, load, friction

mathtiger6 2021-08-31 20:14:31


friction from the wall and force from the hinge

Hiboy 2021-08-31 20:14:31


Normal force, friction force

EthanTLee23 2021-08-31 20:14:31


Friction and tension if the cam is massless

jspiepuppy 2021-08-31 20:14:31


gravitational force, tension

No_u_ 2021-08-31 20:14:31


tension of the rope, friction from the wall

Yufanwang 2021-08-31 20:14:31


Normal force from the wall, normal force from the hinge, frictional force at the wall

bfan0805 2021-08-31 20:14:31


friction, gravitational, force from load, force from hinge, normal force

mathtiger6 2021-08-31 20:14:31


friction and normal force from the wall and force from the hinge

meichenlaub 2021-08-31 20:14:36

Typesetting math: 18%


meichenlaub 2021-08-31 20:14:40
The cam experiences a friction force F f upward, a normal force N from the wall to the left, and a force from the hinge, H, which we
know little about so far.

meichenlaub 2021-08-31 20:15:21


There's no direct gravitational force worth speaking of on the friend; we expect that the load (and therefore the force from the
hinge) will be so much greater than the weight of the friend that we can ignore it.

meichenlaub 2021-08-31 20:15:36


The friction force must obey

F f ≤ μN.

meichenlaub 2021-08-31 20:15:44


And what does F f need to be equal to in order to hold the device up?

Hiboy 2021-08-31 20:17:00


Typesetting math: 18%
W/2
Zian2006 2021-08-31 20:17:00
W
2

EL2024 2021-08-31 20:17:00


W

Nivek 2021-08-31 20:17:00


W/2

EL2024 2021-08-31 20:17:00


W/2

AOPS81619 2021-08-31 20:17:00


W
at least
2

EthanTLee23 2021-08-31 20:17:00


W/2

meichenlaub 2021-08-31 20:17:06


There's a force F f on each cam, and together the need to be equal in magnitude to the force W pulling down. So

meichenlaub 2021-08-31 20:17:09

1
Ff = W.
2

meichenlaub 2021-08-31 20:17:16


This tells us

W
μ≥ .
2N

meichenlaub 2021-08-31 20:17:31


Our goal is to find the minimum μ. From just that equation, can we tell what μ is minimum?

Siddhartha.basu 2021-08-31 20:18:36


nope

mathisawesome01 2021-08-31 20:18:36


No, we need the normal force

EthanTLee23 2021-08-31 20:18:36


No, since we don't know N

Nivek 2021-08-31 20:18:36


no we dont know N

DGA_Student_1 2021-08-31 20:18:36


no

mathtiger6 2021-08-31 20:18:36


no

Hiboy 2021-08-31 20:18:36


No, we don’t know the normal force

taniaban 2021-08-31 20:18:36


not really

meichenlaub 2021-08-31 20:18:50


No, we're basically stuck. We have a formula that relies on knowing the ratio of W / N. Unfortunately, W and N are both unknowns in
this problem.
Typesetting math:For
18%any given W, we can find some N such that, if we assume the normal force is N, the cam can stick.
meichenlaub 2021-08-31 20:19:00
That doesn't mean any μ can work! We don't know that the device can actually generate the normal force N required in that
equation.

meichenlaub 2021-08-31 20:19:17


Let's try a thought experiment. Look back at our free body diagram. Suppose the normal force were very large and the friction
force was small. What would happen to the cam?

AOPS81619 2021-08-31 20:20:37


it would slip down

vkrishnanaz 2021-08-31 20:20:37


it would slip

DGA_Student_1 2021-08-31 20:20:37


it would start to rotate clockwise

A-Aban 2021-08-31 20:20:37


it would close?

theallpro1 2021-08-31 20:20:37


it would rotate left

Siddhartha.basu 2021-08-31 20:20:37


it would fall down

theshuklas 2021-08-31 20:20:37


It would collapse inward

Latnor 2021-08-31 20:20:37


be pushed away from the wall and fall

mathtiger6 2021-08-31 20:20:37


it would slide down

Hiboy 2021-08-31 20:20:37


The cam would collapse?

HappiHippo 2021-08-31 20:20:37


it would slip out?

taniaban 2021-08-31 20:20:37


would it slide down the crag

meichenlaub 2021-08-31 20:20:47


The normal force pushes from below the hinge, and tries to rotate the hinge clockwise. If the normal force were much larger than
the friction force, it would win this battle and start rotating the hinge.

meichenlaub 2021-08-31 20:21:01


What physics concept does this suggest we bring into the conversation?

mathisawesome01 2021-08-31 20:21:26


torque

DGA_Student_1 2021-08-31 20:21:26


torque

rohitk14 2021-08-31 20:21:26


torque

mathtiger6 2021-08-31 20:21:26


torque

A-Aban 2021-08-31 20:21:26


Typesetting math: 18%
torque
EL2024 2021-08-31 20:21:26
torque

Hiboy 2021-08-31 20:21:26


torque

taniaban 2021-08-31 20:21:26


torque

Hershey806 2021-08-31 20:21:26


torque

EthanTLee23 2021-08-31 20:21:26


Torque

AOPS81619 2021-08-31 20:21:26


torque

theallpro1 2021-08-31 20:21:26


torque

Bossman5960 2021-08-31 20:21:26


Torque

meichenlaub 2021-08-31 20:21:31


Let's try again with torque.

meichenlaub 2021-08-31 20:21:35


Torques

meichenlaub 2021-08-31 20:21:38


We'll take the hinge as our axis about which to calculate torques. Why might this be a good idea?

olive0827 2021-08-31 20:22:53


forces at that point go to 0 torque

Hershey806 2021-08-31 20:22:53


hinge force is complicated

DGA_Student_1 2021-08-31 20:22:53


because then the torque from the hinge force is zero

theallpro1 2021-08-31 20:22:53


we can ignore H

Nivek 2021-08-31 20:22:53


dont need to consider the weird forces at the hinge

AOPS81619 2021-08-31 20:22:53


Because the hinge is supposed to be fixed

bfan0805 2021-08-31 20:22:53


hinge force doesnt matter

HappiHippo 2021-08-31 20:22:53


because then we can ignore the force H

piece_of_pi 2021-08-31 20:22:53


cancels out torque from the hinge

Pleaseletmewin 2021-08-31 20:22:53


we don't know anything about H so we don't want to involve it

ethanchen323 2021-08-31 20:22:53


thats where the thing rotates
Typesetting math: 18%
PhysKid11 2021-08-31 20:22:53
*also because the torque from the hinge force becomes 0

meichenlaub 2021-08-31 20:23:02


We don't know the force H from the hinge and we aren't completely sure how to figure it out. But if the hinge is also the axis, the
torque from the force H is zero, no matter how large it is or what direction it points. That simplifies matters for us.

meichenlaub 2021-08-31 20:23:10


What needs to be true about the sum of the normal force from the wall and the friction force so that the cam is in equilibrium (that
it doesn't move)? Can you state your answer in terms of a geometric property of this vector sum?

mathisawesome01 2021-08-31 20:24:58


the vector sum of those forces must be in the direction of the contact point from the hinge (for 0 torque)

AOPS81619 2021-08-31 20:24:58


It should point towards the hinge

PhysKid11 2021-08-31 20:24:58


the vector sum must point towards the hinge

DGA_Student_1 2021-08-31 20:24:58


Their vector sum points along the line connecting the point of contact to the hinge

ethanchen323 2021-08-31 20:24:58


the vector sum of those forces have to point towards the hinge

meichenlaub 2021-08-31 20:25:10


We need the sum of these two forces to exert no torque. The torque is τ = r mF, where r m is the moment arm for the torque and F is
the sum of the forces.

meichenlaub 2021-08-31 20:25:14


The forces are definitely not zero, so it must be the case that the moment arm is zero. And for that to happen, the vector sum of
the friction and the normal forces must point directly towards the hinge.

meichenlaub 2021-08-31 20:25:17

Typesetting math: 18%


meichenlaub 2021-08-31 20:25:29
What does that give you for the minimum μ ?

Pleaseletmewin 2021-08-31 20:26:34


tanθ

mathtiger6 2021-08-31 20:26:34


tanθ

mathisawesome01 2021-08-31 20:26:34


The minimum μ is tanθ

DGA_Student_1 2021-08-31 20:26:34


μ min = tanθ

bfan0805 2021-08-31 20:26:34


tanθ
Typesetting math: 18%
EthanTLee23 2021-08-31 20:26:34
μ=tan(\theta)

Zian2006 2021-08-31 20:26:34


μ = tanθ

Hiboy 2021-08-31 20:26:34


u=tan(theta)

Bossman5960 2021-08-31 20:26:34


μ = tanθ

HappiHippo 2021-08-31 20:26:34


tanθ again?

meichenlaub 2021-08-31 20:26:39


If vector sum of the friction and normal forces points directly towards the hinge, then

Ff
tanθ = .
N

meichenlaub 2021-08-31 20:26:47


Since we're right at the slipping point, F f = μN, and we have

tanθ = μ min.

meichenlaub 2021-08-31 20:26:56


Well, there's a familiar answer! But this time, it came from torques.

meichenlaub 2021-08-31 20:27:13


This is quite neat - once we switched to torques, the weight W didn't even come into our calculations. This means the friend can
actually support any weight. Even if the climber falls a long way and therefore jerks very hard on the rope at the bottom of their fall,
the friend won't slip due to friction.

meichenlaub 2021-08-31 20:27:33


(Of course, in the real world, the friend could break, or the rock could break, etc. But as long as our model of friction is correct,
pulling harder on the rope just results in more normal force and more friction, so that no amount of pulling on the rope can
overcome friction.)

meichenlaub 2021-08-31 20:27:48


If you were climbing and using a device like this, would you want a low θ or a high θ if you weren't sure how large the friction
coefficient with the rock was?

DGA_Student_1 2021-08-31 20:29:02


low

EL2024 2021-08-31 20:29:02


low theta

Hershey806 2021-08-31 20:29:02


low

AOPS81619 2021-08-31 20:29:02


low theta

taniaban 2021-08-31 20:29:02


low

Zian2006 2021-08-31 20:29:02


Low

Nivek 2021-08-31 20:29:02


low
Typesetting math: 18%
theshuklas 2021-08-31 20:29:02
Low

harinsrikanth 2021-08-31 20:29:02


low θ

EthanTLee23 2021-08-31 20:29:02


You would want a low theta

HappiHippo 2021-08-31 20:29:02


a really low θ

bfan0805 2021-08-31 20:29:02


low

Latnor 2021-08-31 20:29:02


low

meichenlaub 2021-08-31 20:29:08


It's probably better to have a low θ. A low θ results in a lower μ min, meaning you're more likely to have a coefficient of friction
higher than the minimum.

meichenlaub 2021-08-31 20:29:36


If you thought that a high θ leads to a high μ, remember that it's an equation for the minimum μ. We don't want a high minimum μ !

meichenlaub 2021-08-31 20:29:46


However, a low θ does result in higher normal forces, so if you're worried about the device or the rock breaking due to too much
force, a large θ would be better.

AOPS81619 2021-08-31 20:30:02


what happens when θ = 0? wouldn't our formula give μ = 0?

meichenlaub 2021-08-31 20:30:14


Yes, and infinite normal force! Clearly the model breaks down.

meichenlaub 2021-08-31 20:30:26


I promise, μ = tanθ isn't the answer to every problem (just most of them.)

meichenlaub 2021-08-31 20:30:41


Let's take a 5 minute break here and then work some more problems.

meichenlaub 2021-08-31 20:36:15


Okay, let's begin again. The next problem is more involved.

meichenlaub 2021-08-31 20:36:18


Consider a solid wooden cylinder with radius R. A smaller cylindrical hole of radius r is drilled out of the cylinder and replaced with
a metal cylinder with higher density. The center of the metal cylinder is a distance d from the center of the wooden cylinder.

meichenlaub 2021-08-31 20:36:22

Typesetting math: 18%


meichenlaub 2021-08-31 20:36:40
The cylinder is placed on a slope with an angle θ from the horizontal that can be varied. It is first set so that θ = 0 and the cylinder
comes to rest. Then the slope is slowly raised to θ = θ f. As the slope rises, the cylinder rotates through an angle ϕ, coming to rest
at this angle when θ = θ f.

meichenlaub 2021-08-31 20:36:58


Find the distance b from the center of the cylinder to its center of mass as a function of θ and ϕ. Assume the cylinder cannot slip
on the incline. (IPhO 2016 Q1, adapted)

meichenlaub 2021-08-31 20:37:14


Let's understand what this question is asking. When the slope is flat so θ = 0, where is the metal part of the cylinder?

theallpro1 2021-08-31 20:38:25


at the bottom

Hiboy 2021-08-31 20:38:25


At the bottom

Pokemon2 2021-08-31 20:38:25


right below the center of the larger cylinder?

A-Aban 2021-08-31 20:38:25


on the bottom

AOPS81619 2021-08-31 20:38:25


On the bottom

EthanTLee23 2021-08-31 20:38:25


The center of the metal part is below the center of the whole cylinder

mathtiger6 2021-08-31 20:38:25


under the center of the wooden cylinder

Latnor 2021-08-31 20:38:25


bottom

mathtiger6 2021-08-31 20:38:25


Typesetting math: 18%
directly under the center of the wooden cylinder
mathisawesome01 2021-08-31 20:38:25
bottom

mathisawesome01 2021-08-31 20:38:25


facing down

meichenlaub 2021-08-31 20:38:29


The metal part is heavier. The heavier parts tend to fall down as low as possible, so the weight is near the bottom. That means the
metal cylinder is directly below the center of the wooden cylinder.

meichenlaub 2021-08-31 20:38:34

meichenlaub 2021-08-31 20:38:46


As we tilt the angle of the slope up, the cylinder rolls. Suppose we lift the right hand side of the slope, so the slope is up and to the
right. Which direction does the cylinder roll?

Latnor 2021-08-31 20:39:18


left

rohitk14 2021-08-31 20:39:18


left

Yufanwang 2021-08-31 20:39:18


Left

ethanchen323 2021-08-31 20:39:18


counterclockwise

mathtiger6 2021-08-31 20:39:18


left

PhysKid11 2021-08-31 20:39:18


down left

A-Aban 2021-08-31 20:39:18


to the left

ishaansiyer 2021-08-31 20:39:18


left

Bossman5960 2021-08-31 20:39:18


the left

Pleaseletmewin 2021-08-31 20:39:18


left

meichenlaub 2021-08-31 20:39:23


The cylinder rolls downhill, to the left.

meichenlaub 2021-08-31 20:39:25

Typesetting math: 18%


meichenlaub 2021-08-31 20:39:35
It seems a little weird that the cylinder can be in equilibrium when resting on an inclined plane, especially since it's round and free
to roll! What force is holding it up?

Z_Lu 2021-08-31 20:40:19


Friction

Pokemon2 2021-08-31 20:40:19


friction

AOPS81619 2021-08-31 20:40:19


Normal force

Nivek 2021-08-31 20:40:19


normal

vkrishnanaz 2021-08-31 20:40:19


normal and frictional forces

EL2024 2021-08-31 20:40:19


normal force?

jspiepuppy 2021-08-31 20:40:19


friction?

harinsrikanth 2021-08-31 20:40:19


friction

EthanTLee23 2021-08-31 20:40:19


Friction

HappiHippo 2021-08-31 20:40:19


normal
Typesetting math: 18%
HappiHippo 2021-08-31 20:40:19
and friction

Pleaseletmewin 2021-08-31 20:40:19


friction

mathtiger6 2021-08-31 20:40:19


friction

mathisawesome01 2021-08-31 20:40:19


friction

meichenlaub 2021-08-31 20:40:23


There's a friction force from the plane. This is why the cylinder "cannot slip on the incline", to quote the problem.

meichenlaub 2021-08-31 20:40:30


The net torque on the cylinder must be zero. What axis do you think will make it easiest to analyze the torque?

mathtiger6 2021-08-31 20:41:19


through the contact point

AOPS81619 2021-08-31 20:41:19


the contact point with the plane

mathisawesome01 2021-08-31 20:41:19


point of contact

Shadow_Heist 2021-08-31 20:41:19


the contact point of the cylinder and the slope

Yufanwang 2021-08-31 20:41:19


The contact point

DGA_Student_1 2021-08-31 20:41:19


axis passing through point of contact, perpendicular to plane of page

EthanTLee23 2021-08-31 20:41:19


The axis around the contact point, since we don't know the friction

Nivek 2021-08-31 20:41:19


touching point

meichenlaub 2021-08-31 20:41:25


Let's choose the point of contact between the cylinder and the plane. There are three forces - gravitational force, the normal force,
and the friction force. If we choose the contact point, then neither the normal force nor the friction force can exert any torque,
since a force never exerts torque about the point where it acts.

meichenlaub 2021-08-31 20:41:40


Of course, we could choose another axis, like the center of mass or the center of the wooden sphere. However, with those axes,
friction exerts a torque, and we don't even know what the value of the friction force is. You might want to try solving the problem
with one of those choices after class to see exactly how the calculations get more involved, but for our time together let's continue
with the point of contact choice for our axis.

meichenlaub 2021-08-31 20:42:17


Given that the normal force and the friction force exert no torque about the contact, where must the center of mass of the system
be?

DGA_Student_1 2021-08-31 20:43:13


directly above the point of contact

Hershey806 2021-08-31 20:43:13


directly above the contact point

taniaban 2021-08-31 20:43:13


directly above the contact point
Typesetting math: 18%
theallpro1 2021-08-31 20:43:13
directly above the contact point

mathtiger6 2021-08-31 20:43:13


directly above the contact point

Square_264 2021-08-31 20:43:13


vertically above the contact

Ymerej 2021-08-31 20:43:13


above the point of contact?

HappiHippo 2021-08-31 20:43:13


right above the contact point

Shadow_Heist 2021-08-31 20:43:13


right above the point of contact

Nivek 2021-08-31 20:43:13


right over the contact point

EL2024 2021-08-31 20:43:13


vertical line off the point of contact?

meichenlaub 2021-08-31 20:43:19


The only other force on the cylinder is the gravitational force. Since there must be no net torque, the gravitational force must exert
no torque, so it must have zero moment arm. This means it needs to be directly above the contact point.

meichenlaub 2021-08-31 20:43:27

meichenlaub 2021-08-31 20:43:37


The dotted
Typesetting line18%
math: still
has length d, and goes from the center of the cylinder to the center of the metal piece. The dot refers to the
center of mass of the system, which is a distance b along the dotted line.)
meichenlaub 2021-08-31 20:43:56
The dashed line must be vertical. That means that these two red distances are the same.

meichenlaub 2021-08-31 20:44:02

meichenlaub 2021-08-31 20:44:10


With that, can you find b in terms of R, θ, and ϕ ?

meichenlaub 2021-08-31 20:44:36


Hint: how long is the top segment in terms of b and ϕ ? How long is the bottom segment in terms of R and θ ?

mathisawesome01 2021-08-31 20:47:04


Rsinθ
b=
sinϕ

bfan0805 2021-08-31 20:47:04


Rsin θ
b=
sin ϕ

Pleaseletmewin 2021-08-31 20:47:04


Rsin θ
b= sin ϕ

HappiHippo 2021-08-31 20:47:04


Rsinθ
b=
sinϕ

EL2024 2021-08-31 20:47:04


b=Rsin(theta)/sin(phi)

Hershey806 2021-08-31 20:47:04


Rsin θ
b = sin ϕ math: 18%
Typesetting
meichenlaub 2021-08-31 20:47:19
The higher of those red segments has length bsinϕ.

meichenlaub 2021-08-31 20:47:31


To find the length of the lower segment, we draw in this line:

meichenlaub 2021-08-31 20:47:32

meichenlaub 2021-08-31 20:47:38


The red dotted line is perpendicular to the slope, which tells us that it makes an angle θ with the vertical. It has length R, so the
lower horizontal red segment has length Rsinθ.

meichenlaub 2021-08-31 20:47:56


The two red segments must be equal, so

bsinϕ = Rsinθ

or solving for b,
\

meichenlaub 2021-08-31 20:48:41

Rsinθ
b=
sinϕ

meichenlaub 2021-08-31 20:48:56


This is the first part of an IPhO problem from 2016! (The rest of the problem looks at the moment of inertia of the cylinder. By the
end of the problem, it's possible to determine everything about the metal cylinder using only measurements made on the cylinder
itself, even if the metal is entirely inside the wood and can't be seen.)
Typesetting math: 18%
meichenlaub 2021-08-31 20:49:11
Energy

meichenlaub 2021-08-31 20:49:14


In the previous problem, we said that when the cylinder is at rest on a flat surface, the metal part is directly below the center of
mass. Can you explain that in terms of the energy of the cylinder?

mathtiger6 2021-08-31 20:50:06


lowest potential energy

HappiHippo 2021-08-31 20:50:06


thats the situation with the lowest potential energy

Pokemon2 2021-08-31 20:50:06


that configuration has a minimum potential

DGA_Student_1 2021-08-31 20:50:06


The potential energy of the cylinder has to be minimum at its equilibrium position.

EthanTLee23 2021-08-31 20:50:06


The potential energy of the system is at a minimum

A-Aban 2021-08-31 20:50:06


that's its lowest potential energy (center of mass is lowest)

Zian2006 2021-08-31 20:50:06


It is at a minimum

EL2024 2021-08-31 20:50:06


its the lowest potential energy state

meichenlaub 2021-08-31 20:50:10


Rotating the cylinder until the metal part is as low as possible minimized the cylinder's potential energy. Recall from the handout
that stable equilibria occur at states with the lowest potential energy.

meichenlaub 2021-08-31 20:50:20


Let's briefly return to this picture:

meichenlaub 2021-08-31 20:50:22

Typesetting math: 18%


meichenlaub 2021-08-31 20:50:30
The cylinder is at equilibrium. That means it should be at its state of lowest potential energy. But we could just roll the cylinder
down the incline to the left a little. Wouldn't that result in potential energy that's even lower?

Pokemon2 2021-08-31 20:51:27


no since the denser part would move up

Shadow_Heist 2021-08-31 20:51:27


no, the metal cylinder would be higher

DGA_Student_1 2021-08-31 20:51:27


no, because the metal cylinder would move up due to the rotation

Pokemon2 2021-08-31 20:51:27


making the net change in the center of mass upwards

Hiboy 2021-08-31 20:51:27


No, as the center of mass rises

bfan0805 2021-08-31 20:51:27


it would increase the gpe of the metal mass

theallpro1 2021-08-31 20:51:27


the metal part would go up, causing a net increase in potential energy

mathtiger6 2021-08-31 20:51:27


no, the metal rises higher

EL2024 2021-08-31 20:51:27


wouldn't that raise the center of mass if you rolled it to the left though

Nivek 2021-08-31 20:51:27


it involves increasing the energy first before it can decrease
Typesetting math: 18%
taniaban 2021-08-31 20:51:27
the metal goes up though

Latnor 2021-08-31 20:51:27


the heavier part would be higher

meichenlaub 2021-08-31 20:51:33


Nope! In fact, rotating the cylinder a little bit would raise the center of mass a little, resulting in higher potential energy.

meichenlaub 2021-08-31 20:51:39


Of course, if we rolled the entire cylinder one whole rotation to the left, it would be lower and the potential energy would go down.
But systems find equilibria at local minima of potential energy. Although this would lower the potential energy of the cylinder, the
potential energy would go up before it went down. The system simply finds an equilibrium where any small displacement increases
the potential energy.

meichenlaub 2021-08-31 20:51:59


Let's see how this works in a little more detail.

meichenlaub 2021-08-31 20:52:01


Suppose we let the cylinder rotate through an angle dϕ down the incline. Does the geometric center of the cylinder go up or down?

Pokemon2 2021-08-31 20:52:32


down

Hiboy 2021-08-31 20:52:32


down

HappiHippo 2021-08-31 20:52:32


down

DGA_Student_1 2021-08-31 20:52:32


down

EthanTLee23 2021-08-31 20:52:32


Down

Pleaseletmewin 2021-08-31 20:52:32


down

EL2024 2021-08-31 20:52:32


down

Zian2006 2021-08-31 20:52:32


Down

bfan0805 2021-08-31 20:52:32


down

Latnor 2021-08-31 20:52:32


down

mathtiger6 2021-08-31 20:52:32


down

Nivek 2021-08-31 20:52:32


down

A-Aban 2021-08-31 20:52:32


down

mathisawesome01 2021-08-31 20:52:32


goes down

meichenlaub 2021-08-31 20:52:36


It goes down, since the cylinder is going downhill, rotating counterclockwise.
Typesetting math: 18%
meichenlaub 2021-08-31 20:52:41
How about the center of mass, relative to the geometric center. Does that go up or down?

Latnor 2021-08-31 20:52:59


up

Zian2006 2021-08-31 20:52:59


Up

Pokemon2 2021-08-31 20:52:59


moves up

Nivek 2021-08-31 20:52:59


up

HappiHippo 2021-08-31 20:52:59


up

Siddhartha.basu 2021-08-31 20:52:59


that goes up

mathtiger6 2021-08-31 20:52:59


up

taniaban 2021-08-31 20:52:59


up

Yufanwang 2021-08-31 20:52:59


Up

mathisawesome01 2021-08-31 20:52:59


up

A-Aban 2021-08-31 20:52:59


up, because metal is denser

DGA_Student_1 2021-08-31 20:52:59


up

Hiboy 2021-08-31 20:52:59


up

PhysKid11 2021-08-31 20:52:59


that goes up

meichenlaub 2021-08-31 20:53:04


The cylinder rotates counterclockwise. That sort of rotate raises the metal piece up higher, raising the center of mass relative to
the geometric center.

meichenlaub 2021-08-31 20:53:23


If the cylinder is in equilibrium, what must happen with the two height changes we just discussed?

EL2024 2021-08-31 20:54:12


they are equal

taniaban 2021-08-31 20:54:12


equal

Hiboy 2021-08-31 20:54:12


equal to one another

jspiepuppy 2021-08-31 20:54:12


they must balance eachother out

Bossman5960 2021-08-31 20:54:12


must balance eachother?
Typesetting math: 18%
DGA_Student_1 2021-08-31 20:54:12
they must cancel, as the virtual work over an infinitesimal displacement is zero for a system in equilibrium

meichenlaub 2021-08-31 20:54:16


They're going to have to cancel. The center of mass falls due to the entire cylinder going downhill, but rises due to rotation of the
cylinder. Combining these effects, we should get zero change in the height of the center of mass at equilibrium. Let's make sure
this works.

meichenlaub 2021-08-31 20:54:36


How much does the geometric center of the cylinder rise when it rotates through an angle dϕ ?

meichenlaub 2021-08-31 20:55:06

Pokemon2 2021-08-31 20:56:18


Rdϕsin(θ)

EL2024 2021-08-31 20:56:18


Rd(phi)sin(theta)

Bossman5960 2021-08-31 20:56:18


Rdϕsinθ

mathtiger6 2021-08-31 20:56:18


Rdϕsinθ

EthanTLee23 2021-08-31 20:56:18


Rdϕsinθ

Hiboy 2021-08-31 20:56:18


R*dphi*sin(theta)

DGA_Student_1
Typesetting math: 18% 2021-08-31 20:56:18
− Rsinθ dϕ
meichenlaub 2021-08-31 20:56:35
The geometric center moves a distance Rdϕ in a line parallel to the surface of the incline, so it rises a distance

meichenlaub 2021-08-31 20:56:38

dy geometric center = − Rsinθdϕ.

meichenlaub 2021-08-31 20:56:48


And how much does the center of mass rise, relative to the geometric center of the cylinder? (Use b in your answer.)

meichenlaub 2021-08-31 20:58:29

Pleaseletmewin 2021-08-31 20:59:37


bsinϕdϕ

Bossman5960 2021-08-31 20:59:37


bdϕsinϕ

taniaban 2021-08-31 20:59:37


bdϕsinϕ

mathtiger6 2021-08-31 20:59:37


bdϕsinϕ

DGA_Student_1 2021-08-31 20:59:37


bsinϕ dϕ

Zian2006 2021-08-31 20:59:37


Oops bsinϕdϕ

mathisawesome01 2021-08-31 20:59:37


Typesetting math: 18%
bsinϕdϕ
Hiboy 2021-08-31 20:59:37
bsin(phi)dphi

EthanTLee23 2021-08-31 20:59:37


bdϕsinϕ

meichenlaub 2021-08-31 20:59:51


The center of mass moves a distance bdϕ. It moves along a line inclined to the horizontal at angle ϕ, so it rises

meichenlaub 2021-08-31 21:00:02

dy com, relative = bsinϕdϕ.

meichenlaub 2021-08-31 21:00:11


Given that bsinϕ = Rsinθ, (which we calculated using torques) how much does the height of the center of mass change when we
roll the cylinder by an infinitesimal angle dθ ?

taniaban 2021-08-31 21:01:34


0

mathtiger6 2021-08-31 21:01:34


0

DGA_Student_1 2021-08-31 21:01:34


zero

bfan0805 2021-08-31 21:01:34


0

mathisawesome01 2021-08-31 21:01:34


0

Bossman5960 2021-08-31 21:01:34


0

HappiHippo 2021-08-31 21:01:34


0

meichenlaub 2021-08-31 21:01:44


The center of mass doesn't rise or fall at all. Its change in height is

dy com = dy geometric center + dy com, relative = − Rsinθ + bsinϕ = 0.

meichenlaub 2021-08-31 21:01:57


Weird! We just said that any small displacement leads to raising the center of mass and increasing the energy. But now we've
shown that a small displacement leads to zero change in the height of the center of mass. Can anyone explain what's going on?

mathisawesome01 2021-08-31 21:02:59


we found to the first order, second order is where the increase shows up

mathtiger6 2021-08-31 21:02:59


we did them in first-order

Zian2006 2021-08-31 21:02:59


dE
It is at the bottom of the energy curve, so dx = 0

meichenlaub 2021-08-31 21:03:06


Suppose you're at the bottom of a hill

meichenlaub 2021-08-31 21:03:09

Typesetting math: 18%


meichenlaub 2021-08-31 21:03:11
Then moving either left or right takes you higher.

meichenlaub 2021-08-31 21:03:14


dy
On the other hand, the derivative is zero there, so = 0, which would imply dy = 0.
dx

meichenlaub 2021-08-31 21:03:21


The point is that a step left or right takes you higher, but only to second order. That is, if your step size is dx, the amount you rise is
proportional to dx 2. That means it doesn't show up in a derivative.

meichenlaub 2021-08-31 21:03:34


In the example we just worked, rotation by an angle dθ raises the center of mass, but by an amount proportional to dθ 2. This
dU
amount doesn't show up when we look at the derivative of potential energy, .

meichenlaub 2021-08-31 21:04:01


The upshot is that we can either require that the potential energy U(x) of a system is minimized at equilibrium, or we can look for
dU dU
places where = 0. These give the same thing. (Except that places where = 0 can also find maxima, etc. These are points of
dx dx
equilibrium, too, just usually not that physically relevant because they are unstable. A lot of olympiad problems ask you to
distinguish stable and unstable equilibria, so knowing both of these methods is useful.)

meichenlaub 2021-08-31 21:04:31


Four long and four half-as-long rods are hinged to each other, forming three identical rhombi. One end of the contraption is hinged
to a ceiling. The other end is attached to a mass m. The hinge next to the mass is connected to the hinge above by a massless
rope. Find the tension T in the rope. The rods are massless and rigid.

meichenlaub 2021-08-31 21:04:35

meichenlaub 2021-08-31 21:04:48


This looks kind of complicated. There are lots of rods in there, oriented at some completely unidentified angles, and it might be
kind of a mess to figure out. But let's give an energy-based approach a try.

meichenlaub 2021-08-31 21:04:54


Suppose we let the weight fall a small distance dy. Then of course its gravitational potential energy goes down. But we are
seeking to analyze an equilibrium position and therefore want the potential energy change dU to be zero when we do this. How is
this possible? What other potential energy will we have to account for?
Typesetting math: 18%
mathtiger6 2021-08-31 21:05:47
potential energy in the rope

mathisawesome01 2021-08-31 21:05:47


rope

DGA_Student_1 2021-08-31 21:05:47


elastic potential energy

Zian2006 2021-08-31 21:05:47


Rope is streched

EthanTLee23 2021-08-31 21:05:47


Elastic potential

mathisawesome01 2021-08-31 21:05:47


potential energy in the rope, due to tension

meichenlaub 2021-08-31 21:05:53


We'll need to think about potential energy in the rope - that's the only other place it can be.

meichenlaub 2021-08-31 21:05:57


So if we stretch the rope by an amount ds, how much do we change its potential energy?

meichenlaub 2021-08-31 21:07:14


I'm getting a lot of answers with k in them, but there's no k in this problem!

Zian2006 2021-08-31 21:07:54


Tds

mathtiger6 2021-08-31 21:07:54


Tds because of work

Yufanwang 2021-08-31 21:07:54


Tds

Dodgers66 2021-08-31 21:07:54


Tds

meichenlaub 2021-08-31 21:08:05


The rope experiences tension T. To stretch it, we'd need to pull with a force T, doing work Tds. So the potential energy change is

meichenlaub 2021-08-31 21:08:12

dU string = Tds.

meichenlaub 2021-08-31 21:08:26


If the system is in equilibrium, then dU total = 0 for a small displacement of the mass downward. How can we write dU total in terms
of m, g, dy, and ds ?

Pokemon2 2021-08-31 21:09:28


Tds-mgdy

mathtiger6 2021-08-31 21:09:28


dU total = − mgdy + Tds

taniaban 2021-08-31 21:09:28


T ds - mg dy

Zian2006 2021-08-31 21:09:28


− mgdy + Tds

Dodgers66 2021-08-31 21:09:28


Tds − mgdy

Typesetting
Hershey806 math: 2021-08-31
18% 21:09:28
Tds − mgdy

meichenlaub 2021-08-31 21:09:33


It's

meichenlaub 2021-08-31 21:09:34

dU total = − mgdy + Tds = 0

meichenlaub 2021-08-31 21:09:48


We can solve that for T to get

meichenlaub 2021-08-31 21:09:52

mgdy
T= .
ds

meichenlaub 2021-08-31 21:09:56


What do you get when you simplify that further?

Dodgers66 2021-08-31 21:11:08


3mg

mathisawesome01 2021-08-31 21:11:08


T = 3mg since dy = 3ds

Zian2006 2021-08-31 21:11:08


dy
ds
=3

EL2024 2021-08-31 21:11:08


is ds equal to 1/3dy?

mathtiger6 2021-08-31 21:11:08


dy / ds = 3 because there are three parts to extend the contraption --> T = 3mg

theallpro1 2021-08-31 21:11:08


3mg

HappiHippo 2021-08-31 21:11:08


T = 3mg?

taniaban 2021-08-31 21:11:08


3mg

Zian2006 2021-08-31 21:11:08


T = 3mg

meichenlaub 2021-08-31 21:11:14


The length of the rope, s, is one third of the distance from the mass to the top of the contraption, which we can call ℓ. That gives
us

1
s= ℓ.
3

meichenlaub 2021-08-31 21:11:17


If the mass falls by a distance dy, then ℓ increases by dy, so

dℓ = dy

Substituting in dℓ = 3ds and solving for ds,

Typesetting math: 18%


1
ds = dy.
3

meichenlaub 2021-08-31 21:11:23


Putting that into the equation for T,

T = 3mg.

meichenlaub 2021-08-31 21:11:25


The interesting thing is that it doesn't matter what the angles are between the rods. An energy-based approach shows us that the
tension in the rope follows immediately from the fact that the rope length is just a certain fraction of the contraption length.

meichenlaub 2021-08-31 21:12:04


Next we'll look at something from IPhO 2010.

meichenlaub 2021-08-31 21:12:08


A plastic film is rolled into a cylinder. The potential energy of the cylinder is

meichenlaub 2021-08-31 21:12:11

κ 1
U= A
2 R2
c

meichenlaub 2021-08-31 21:12:18


where

meichenlaub 2021-08-31 21:12:19

κ = bending modulus of the plastic


R c = radius of the cylinder
A = area of the sides of the cylinder (excluding bases)

meichenlaub 2021-08-31 21:12:27


The cylinder is placed between two flat boards which are placed on a scale, allowing us to measure F, the force between the board
and the cylinder, provided by a mass sitting on top the board. The cylinder deforms into a "stadium", a shape whose cross-section
is two half-circles connected by straight segments.

meichenlaub 2021-08-31 21:12:39

meichenlaub 2021-08-31 21:12:54


Find the bending modulus κ as a function of R 0, F, and l, where l is the length of the cylinder in the direction perpendicular to the
cross section.

meichenlaub 2021-08-31 21:13:10


The problem
Typesetting math:also
18% provided this photo of the setup:
meichenlaub 2021-08-31 21:13:13

meichenlaub 2021-08-31 21:13:46


So this problem involves a formula which includes a "bending modulus", something we haven't heard of before. There's also not
really much explanation for where this formula comes from. This is common on olympiad physics problems - you can be given new
and unfamiliar stuff and asked to work with it. The basic principles are those we already know.

meichenlaub 2021-08-31 21:13:52


What principle will we use, given that this whole thing is in equilibrium?

Dodgers66 2021-08-31 21:14:35


virtual work

Pokemon2 2021-08-31 21:14:35


virtual work?

DGA_Student_1 2021-08-31 21:14:35


the potential energy has to be stationary

EL2024 2021-08-31 21:14:35


virtual work

EthanTLee23 2021-08-31 21:14:35


Statics with energy

Nivek 2021-08-31 21:14:35


minimum potential energy
Typesetting math: 18%
bananaman321 2021-08-31 21:14:35
energy

meichenlaub 2021-08-31 21:14:39


We're given a formula for potential energy, so it seems logical to proceed with an energy approach. The board-film system is in
equilibrium, so we need to minimize the potential energy of the system. So we want dU total = 0.

meichenlaub 2021-08-31 21:14:55


What is the small displacement we'll consider when calculating dU ?

DGA_Student_1 2021-08-31 21:15:49


small vertical displacement

Pokemon2 2021-08-31 21:15:49


small vertical displacement

PhysKid11 2021-08-31 21:15:49


the displacement of the board on which the block rests

mathisawesome01 2021-08-31 21:15:49


small displacement of the board

mathtiger6 2021-08-31 21:15:49


lower the mass

taniaban 2021-08-31 21:15:49


up or down

Yufanwang 2021-08-31 21:15:49


A slight shift in the top flat board

meichenlaub 2021-08-31 21:15:54


For this system, the forces we're interested in are forces that hold the board up on top of the film. These forces stop the board
from falling, so we'll look at a small vertical displacement of the board.

meichenlaub 2021-08-31 21:16:05


What's dU gravitational when we let the board rise a distance dy ?

mathtiger6 2021-08-31 21:17:00


+ mgdy

HappiHippo 2021-08-31 21:17:00


mgdy

Pokemon2 2021-08-31 21:17:00


mgdy?

EthanTLee23 2021-08-31 21:17:00


mgdy

Pleaseletmewin 2021-08-31 21:17:00


mgdy

Hiboy 2021-08-31 21:17:00


dy*m*g

theallpro1 2021-08-31 21:17:00


m g dy

mathisawesome01 2021-08-31 21:17:00


mgdy

taniaban 2021-08-31 21:17:00


mgdy
Typesetting math: 18%
meichenlaub 2021-08-31 21:17:05
It's

meichenlaub 2021-08-31 21:17:07

dU gravitational = mgdy = Fdy.

meichenlaub 2021-08-31 21:17:19


κ 1
Now we need to deal with dU film. We're given U film = A. Can you rewrite that formula for U to get rid of A and use l instead?
2 R2
c

Yufanwang 2021-08-31 21:18:41


A = 2πlR

Hiboy 2021-08-31 21:18:41


A=l*pi*2*R

DGA_Student_1 2021-08-31 21:18:41


πκl
U film = R
c

theallpro1 2021-08-31 21:18:41


κ 2πl
2 Rc

Hiboy 2021-08-31 21:18:41


A=l*2*R

meichenlaub 2021-08-31 21:18:45


The area of the cylinder is A = 2πR cl, so

κ R c2πl l
U film = = κπ .
2 R2 Rc
c

meichenlaub 2021-08-31 21:18:59


When we look at the stadium shape, there are flat parts and round parts. What potential energy is stored in the flat parts?

dagurvich 2021-08-31 21:20:02


0

Zian2006 2021-08-31 21:20:02


0

theallpro1 2021-08-31 21:20:02


0

DGA_Student_1 2021-08-31 21:20:02


zero?

meichenlaub 2021-08-31 21:20:12


Zero! Those parts are not deformed from the original, flat shape of the sheet. Formally, in those parts, the radius of curvature
l
blows up, R c → ∞, so from the equation U film = κπ we see that U film = 0 for those parts.
Rc

meichenlaub 2021-08-31 21:20:31


What is dU film in terms of κ, R 0, and dy ?

mathtiger6 2021-08-31 21:24:09


− κπℓdy / 2R 20

meichenlaub 2021-08-31 21:24:27


Typesetting math: 18%
We can let R c become R 0, since the radius of curvature (of those parts that are curved) is R 0.
meichenlaub 2021-08-31 21:24:30
Then taking the derivative of U film, we get

l
dU film = − πκ 2 dR 0.
R0

meichenlaub 2021-08-31 21:24:45


1
However, we wanted this in terms of dy. Because the height is 2R 0, dR 0 = dy. Therefore
2

meichenlaub 2021-08-31 21:24:55

πκ l
dU film = − dy.
2 R2
0

meichenlaub 2021-08-31 21:25:03


Now what do you get for the answer to the question?

mathtiger6 2021-08-31 21:27:05


2R 20F
κ = πℓ

mathisawesome01 2021-08-31 21:27:05


2FR 2
0
κ=
πl

HappiHippo 2021-08-31 21:27:05


2FR 20
κ=
πl

EthanTLee23 2021-08-31 21:27:05


κ = 2FR 2 / (\pil)

Hershey806 2021-08-31 21:27:05


2
2FR 0
κ= πl

Zian2006 2021-08-31 21:27:05


2
2R 0F
κ= πl

Yufanwang 2021-08-31 21:27:05


2FR 20
κ=
πl

EthanTLee23 2021-08-31 21:27:05


$\kappa=2FR^2/(l\pi)

DGA_Student_1 2021-08-31 21:27:05


2F 2
κ= R
πl 0

EthanTLee23 2021-08-31 21:27:05


2
κ = 2FR / (lπ)

meichenlaub 2021-08-31 21:27:09


Setting the overall energy change to zero,

dU total = 0 = dU gravitational + dU film.

Typesetting math: 18%


meichenlaub 2021-08-31 21:27:15
Substituting in our equation for each of those contributions,

πκ l
0 = Fdy − dy.
2 R2
0

meichenlaub 2021-08-31 21:27:21


Solving for κ,

2
2FR 0
κ= .
πl

meichenlaub 2021-08-31 21:27:29


Unfortunately, we still don't know exactly what κ means. But what are its dimensions?

meichenlaub 2021-08-31 21:27:49


(Recall that dimensions are things like length, mass, etc. Newtons, meters, etc. are units.)

mathisawesome01 2021-08-31 21:28:53


energy

Hiboy 2021-08-31 21:28:53


Force*length

mathisawesome01 2021-08-31 21:28:53


2
ML
T2

taniaban 2021-08-31 21:28:53


energy

Zian2006 2021-08-31 21:28:53


Energy

mathtiger6 2021-08-31 21:28:53


energy

HappiHippo 2021-08-31 21:28:53


energy

HappiHippo 2021-08-31 21:28:53


work

DGA_Student_1 2021-08-31 21:28:53


2 −2
ML T

smhgeo422 2021-08-31 21:28:53


energy?

EL2024 2021-08-31 21:28:53


oh its the same as energy?

mathtiger6 2021-08-31 21:28:53


2 −2
[M][L] [T]

Hershey806 2021-08-31 21:28:53


ML2
T2

meichenlaub 2021-08-31 21:29:02


κ has dimensions of force ⋅ length, or energy, M ⋅ L 2 ⋅ T − 2.

meichenlaub 2021-08-31 21:29:07


Typesetting math: 18%
For now, we can think of κ as setting the scale for the energy needed to bend the sheet into a cylinder whose height and diameter
are similar to each other.

meichenlaub 2021-08-31 21:29:35


Summary

meichenlaub 2021-08-31 21:29:37


When an object is in static equilibrium, there must be no net force on it and no net torque on it.

meichenlaub 2021-08-31 21:29:41


It's advantageous to choose a smart axis when calculating torques. If there's a force whose value we don't know and don't need to
find, choose an axis so that force exerts zero torque.

meichenlaub 2021-08-31 21:29:44


Systems are also in static equilibrium when they're in their minimum potential energy state, or when small changes to the position
of the system cause no first-order change in potential energy. This is equivalent mathematically to the statement about torques
and forces, but may be easier to work with in certain problems.

meichenlaub 2021-08-31 21:29:48


The surface of a fluid in equilibrium is an equipotential surface.

meichenlaub 2021-08-31 21:29:52


The stretching of solids is characterized by their Young's modulus. Solids with a higher Young's modulus are stiffer and harder to
stretch or bend.

meichenlaub 2021-08-31 21:30:13


(There were a couple more problems at the end we didn't have time for, but I'll post the script for those on the message boards.)

meichenlaub 2021-08-31 21:30:15


Credits for today

meichenlaub 2021-08-31 21:30:17


Golden Gate Bridge: By Photograph by D Ramey Logan, CC BY 4.0, https://commons.wikimedia.org/w/index.php?curid=45718213

I-Beam: BThomasHart Photography

Poisson ratio gif: Rod Lakes, "Meaning of Poisson's ratio", http://silver.neep.wisc.edu/~lakes/PoissonIntro.html

Iron Deposit question: Kalda, Jaan. "Mechanics" https://www.ioc.ee/~kalda/ipho/meh_ENG2.pdf

© 2022 Art of Problem Solving


About Us • Contact Us • Terms • Privacy

Copyright © 2022 Art of Problem Solving

Typesetting math: 18%

You might also like